Your-Doctor
Multiple Choice Questions (MCQ)


Quiz Categories Click to expand

Category: Medicine--->Cardiology
Page: 1

Question 1# Print Question

A 60-year-old male patient is receiving aspirin, an angiotensin-converting enzyme inhibitor, nitrates, and a beta-blocker for chronic stable angina. He presents to the ER with an episode of more severe and long-lasting anginal chest pain each day over the past 3 days. His ECG and cardiac enzymes are normal.

Which of the following is the best course of action?

A. Admit the patient and add intravenous digoxin
B. Admit the patient and begin low-molecular-weight heparin
C. Admit the patient for thrombolytic therapy
D. Admit the patient for observation with no change in medication
E. Increase the doses of current medications and follow closely as an outpatient


Question 2# Print Question

You have been asked to evaluate a 42-year-old white male smoker who presented to the emergency department with sudden onset of crushing substernal chest pain, nausea, diaphoresis, and shortness of breath. His initial ECG revealed ST segment elevation in the anteroseptal leads. Cardiac enzymes were normal. The patient underwent emergent cardiac catheterization, which revealed only a 25% stenosis of the left anterior descending (LAD) artery. No percutaneous intervention was performed. Which of the following interventions would most likely reduce his risk of similar episodes in the future?

A. Placement of a percutaneous drug-eluting coronary artery stent
B. Placement of a percutaneous non–drug-eluting coronary artery stent
C. Beginning therapy with an ACE inhibitor
D. Beginning therapy with a beta-blocker
E. Beginning therapy with a calcium-channel blocker


Question 3# Print Question

A 15-year-old student presents to your office on the advice of his football coach. The patient started playing football this year and suffered a syncopal episode at practice yesterday. He reports that he was sprinting with the rest of the team and became light-headed. He lost consciousness and fell to the ground, regaining consciousness within 1 or 2 minutes. He has had no prior episodes of syncope. The patient is adopted and family history unavailable. Physical examination reveals a systolic murmur heard at the left lower sternal border and apex. ECG reveals sinus rhythm with evidence of left ventricular hypertrophy (LVH). What physical examination findings would likely be present? 

A. A systolic ejection murmur heard best at the apex that diminishes with squatting and handgrip, and increases with Valsalva maneuver and standing
B. A systolic murmur with mid to late systolic click heard at the apex. The click and murmur occur earlier in systole with squatting and handgrip and are delayed with Valsalva maneuver and standing
C. A holosystolic murmur heard best at the apex, radiating to the axilla, which increases with squatting and hand grip, and diminishes with Valsalva maneuver and standing
D. A blowing holosystolic murmur heard best at the lower left sternal border which increases with squatting and hand grip and diminishes with Valsalva maneuver and standing
E. A low pitched mid systolic murmur radiating to the carotids


Question 4# Print Question

An 82-year-old white woman is admitted to the hospital for observation after presenting to the emergency department with dizziness. After being placed on a cardiac monitor in the ER, the rhythm strip below was recorded. There is no past history of cardiac disease, diabetes, or hypertension. With prompting, the patient discloses several prior episodes of transient dizziness and one episode of brief syncope in the past. Physical examination is unremarkable. Which of the following is the best plan of care?

A. Reassurance. This is a benign condition, and no direct therapy is needed
B. Reassurance. The patient may not drive until she is symptom free, but otherwise no direct therapy is needed
C. Nuclear cardiac stress testing; treatment depending on results
D. Begin therapy with aspirin
E. Arrange placement of a permanent pacemaker


Question 5# Print Question

One month after hospital discharge for documented myocardial infarction, a 65-year-old man returns to your office concerned about low-grade fever and chest pain. He describes the chest pain as sharp, worse on deep inspiration and better when sitting up. He denies shortness of breath; his lungs are clear to auscultation. On your heart examination you do not appreciate any murmur or rubs. ECG is shown below. Which therapy is most likely to be effective in relieving his chest pain?

A. Antibiotics
B. Warfarin
C. An anti-inflammatory agent
D. Nitrates
E. An anxiolytic




Category: Medicine--->Cardiology
Page: 1 of 9